LSAT and Law School Admissions Forum

Get expert LSAT preparation and law school admissions advice from PowerScore Test Preparation.

 Administrator
PowerScore Staff
  • PowerScore Staff
  • Posts: 8917
  • Joined: Feb 02, 2011
|
#25092
Complete Question Explanation
(The complete setup for this game can be found here: lsat/viewtopic.php?t=4898)

The correct answer choice is (A)

As noted during the setup discussion (lsat/viewtopic.php?f=120&t=4898) of the inference resulting from the combination of the first and second rules, J and N cannot both be assigned ambassadorships, and hence answer choice (A) is correct.
 scai10
  • Posts: 1
  • Joined: Sep 18, 2015
|
#19860
I read question 8 to mean, which two candidates could be the two that are NOT assigned an ambassadorship. I picked A by going through all of the possibilities of the candidates, if the answer choices' candidates were not assigned (KLO, KLN, JNO, etc.).

However, in the answer explanations, it simply says A is correct because J :dblline: N. That's an inference I did see, but I don't understand why because both cannot be chosen, it automatically means that those two could be NOT assigned together.

Or did I misinterpret the question. Does the question actually mean which candidates cannot both be assigned an ambassadorship at the same time? If so, that question is written in a confusing manner!
 Jon Denning
PowerScore Staff
  • PowerScore Staff
  • Posts: 904
  • Joined: Apr 11, 2011
|
#19882
Hey scai,

Thanks for the question, and welcome to the Forum!

This question is just asking what two people could both be unassigned simultaneously (out together, as I think of it). So who could we pull out and still leave three people to satisfy the assignment rules?

The J :dblline: N inference just comes from the first two rules: if J is assigned then so is K, and K and N can't be together. So J brings K, which knocks out N, meaning J and N cannot be together.

Of course, that's only a problem if they're in together; they can both be out without an issue:

..... Kv , Oy , Lz assigned; J and N not assigned

So (A) is correct because with J and N out, you'd have K, O, and L in and that can work!

That also means the other four answers present "out" pairs that are impossible, as follows:

(B) with J and O out, you'd have K and N both in and violate a rule
(C) with K and L out, you'd have J in without K and violate a rule (remember, J needs K!)
(D) with K and N out, you'd again have J in without K and violate a rule
(E) with L and O out, you'd have K and N both in and violate a rule

So even if you were a little uncertain about (A), a quick progression through the remaining options should have allowed you to knock them out pretty easily.

I hope that helps to clarify this question, and also emphasize how important it is to focus on both groups, in and out, when you have Overfunded Grouping games!

Get the most out of your LSAT Prep Plus subscription.

Analyze and track your performance with our Testing and Analytics Package.